Séries numériques

L'étude des séries numériques est une partie importante de programme de PSI, beaucoup de thèmes dépend en partie (ou en totalité) de celui-ci, on peut donner par exemple:

  • L'étude des séries de fonctions : Chercher le domaine de définition, étudier la convergence normale, étudier la suite de reste pour déterminer s'il y a convergence uniforme ou pas.
  • Calcul des moments d'une variable aléatoire discret (espérance, variance ,...).
Il sera alors question de savoir diagnostiquer si une série numérique converge ou diverge, et éventuellement en cas de convergence calculer sa somme.
Dans tout le chapitre, $\K$ désigne $\R$ ou $\C$.

Rappels

Soit $(u_n)_{n \in \N}$ une suite d'éléments de $\K$. On appelle série numérique de terme général $u_n$ la suite de terme général $U_n$, où $U_n=\dsp\sum_{k=0}^n u_k$.
Cette série se note de façon abrégée : $\sum u_n$; $U_n$ s'appelle la somme partielle d'indice $n$ de la série.

Remarque

À toute série est associée, par définition, la suite se ses sommes partielles. Mais, réciproquement, à toute suite $U_n$, on peut associer la série de terme général $u_n$, où : $u_0=U_0$ et $u_n=U_n-U_{n-1}$ pour $n\geqslant 1$.

On dit qu'une série de terme général $u_n$ est convergente si la suite $(U_n)_{n\in \N}$ de ses sommes partielles est convergente.
On note alors $\dsp \sum_{k=0}^{+ \infty}u_k$, appelée somme de la série, la limite de $U_n$, i.e $$\dsp \sum_{k=0}^{+ \infty} u_k = \dsp \lim_{n \to + \infty} \dsp \sum_{k=0}^n u_k.$$ Une série non convergente est dite divergente .

Remarque

L'écriture $\sum u_n$ est une simple notation pour désigner la série de terme général $u_n$. Cependant, l'écriture $\dsp \sum_{k=0}^{+ \infty} u_k$ ne doit, elle, n'être utilisée qu'après avoir démontré la convergence de la série!

Soit $\sum u_n$ une série convergente, et $U=\dsp \sum_{k=0}^{+ \infty} u_k$ sa somme. On appelle reste d'ordre $n$ de cette série l'élément de $\K$ défini par : $R_n = U - U_n = \dsp \sum_{k=n+1}^{+ \infty}u_k$.

Remarque

On a les relations : $\forall n \in \N^*, \ u_n = R_{n-1}-R_n$ et $\dsp \lim_{n \to +\infty} R_n =0$.

Si la série de terme général $u_n$ converge, alors $\dsp \lim_{n \to +\infty}u_n=0$.

Remarque

La réciproque de ce théorème est fausse : on peut avoir $\dsp \lim_{n \to +\infty}u_n=0$ et $\sum u_n$ divergente !

Exemples

  1. L'exemple classique de la série harmonique: $\dsp\sum_{n\in\N^*} \frac1n$ est divergente, cependant, son terme général tend vers $0$ !
  2. On considère $\dsum_{n\geq 0} (\sqrt{n+1}-\sqrt{n})$, on a: $\sqrt{n+1}-\sqrt{n}=\dfrac{1}{\sqrt{n+1}+\sqrt{n}}\tendversN\,0.$
    Pourtant, pour tout $N\geq 0$, on a : $S_N=\dsum_{n=0}^N(\sqrt{n+1}-\sqrt{n})=\sqrt{N+1}\tendvers{N}{\infty}\,\infty.$

Une série dont le terme général ne tend pas vers $0$ est dite grossièrement divergente .

Soient $\sum u_n$ et $\sum v_n$ deux séries convergentes, de sommes respectives $U$ et $V$, et soit $\lambda \in \K$.
Alors la série $\sum (\lambda u_n + v_n)$ est convergente , de somme $\lambda U +V$.

L'ensemble $\mathscr{B}_c$ est un s-e-v de l'e-v des séries d'éléments de $\K$, et l'application qui à une série convergente associe sa somme, est linéaire.

Remarques

  • Si $\sum u_n$ est convergente et $\sum v_n$ divergente, alors $\sum(u_n+v_n)$ est divergente.
  • Si $\sum u_n$ et $\sum v_n$ sont divergentes, on ne peut rien dire a priori de $\sum(u_n+v_n)$.

Soit $\sum u_n$ une série à termes complexes.
$\sum u_n$ est convergente si et seulement si les séries réelles $\sum \re(u_n)$ et $\sum \im(u_n)$ convergent.
Dans ce cas, on a $$\dsp \sum_{k=0}^{+ \infty} u_k = \dsp \sum_{k=0}^{+ \infty} \re(u_k) + \ii \dsp \sum_{k=0}^{+ \infty} \im(u_k).$$

Exemples classiques

Séries géométriques

Rien La série géométrique $\dsp\sum_{n\in\N}q^n$ converge ssi $|q| < 1$ et, dans ce cas : Rien $$\dsp\sum_{n=0}^{+ \infty} q^n= \dfrac1{1-q}\text{ et }R_n=\dfrac{q^{n+1}}{1-q}.$$


Soit $\rho\in ]0,1[,\,\theta\in \R$. Montrer la convergence de $\dsum \rho^n\cos(n\theta)$ (resp. $\dsum \rho^n\sin(n\theta)$) puis calculer leurs sommes.

Correction

On pose $z=\rho\ee^{\ii \theta}\in \C$, on a $\abs{z}=\rho < 1$, d'après la proposition précédente, on a $\dsum z^n$ converge et, $$\begin{array}{lcl} \dsum_{n\geq 0} z^n&=&\dfrac{1}{1-z}=\dfrac{1}{(1-\rho\cos(\theta))-\ii \rho\sin(\theta)}\\ &=&\dfrac{(1-\rho\cos(\theta))+\ii \rho\sin(\theta)}{(1-\rho\cos(\theta))^2+ \rho\sin(\theta)^2}=\dfrac{(1-\rho\cos(\theta))+\ii \rho\sin(\theta)}{\rho^2-2\rho\cos(\theta)+1}\\ &=& \dfrac{(1-\rho\cos(\theta))}{\rho^2-2\rho\cos(\theta)+1}+\ii\dfrac{\rho\sin(\theta)}{\rho^2-2\rho\cos(\theta)+1} \end{array} $$ On en déduit, en utilisant le théorème sur les séries numériques complexes, que les séries $\dsum \rho^n\cos(n\theta)$ et $\dsum \rho^n\sin(n\theta)$ convergent et $$\dsum_{n\geq 0} \rho^n\cos(n\theta)=\re (\dsum z^n)=\dfrac{1-\rho\cos(\theta)}{\rho^2-2\rho\cos(\theta)+1},$$ $$\dsum_{n\geq 0} \rho^n\sin(n\theta)=\im (\dsum z^n)=\dfrac{\rho\sin(\theta)}{\rho^2-2\rho\cos(\theta)+1}.$$

Séries télescopiques

La série télescopique $\dsp\sum_{n\in\N^*}(u_n-u_{n-1})$ converge ssi la suite $(u_n)$ converge,
et, dans ce cas : $$\dsp\sum_{k=1}^{+\infty}(u_k-u_{k-1}) = \dsp\lim_{n\to + \infty}u_n-u_0.$$

Exemple

La série $\dsp\sum_{n=1}^{+ \infty} \dfrac1{n(n+1)}$ converge et de somme $ 1$.

En effet, pour tout $n\in \N^*$, on a $$\dfrac1{n(n+1)}=\dfrac1{n}-\dfrac1{n+1}.$$ Donc, pour $N\geq 1$, on obtient $$\dsum_{n=1}^N\dfrac1{n(n+1)}=\dsum_{n=1}^N\left(\dfrac1{n}-\dfrac1{n+1}\right)=1-\dfrac{1}{N+1}\tendvers{N}{\infty}\,1.$$

Danger

L'écriture $$\xcancel{ \underbrace{\dsp\sum_{n=1}^{+\infty}\dfrac1{n(n+1)}}_{\text{CV}}= \underbrace{\dsp\sum_{n=1}^{+\infty}\dfrac1n}_{\text{DV}} \ - \ \underbrace{\dsp\sum_{n=1}^{+\infty}\dfrac1{n+1}}_{\text{DV}}}$$} n'a aucun sens!!!


Rien

Rapport du jury -- E3A 2022

2. L'erreur la plus fréquente rencontrée est la manipulation de séries divergentes : il vaut toujours mieux manipuler des sommes partielles.

Nous avons trop souvent lu : $$\dsum_{n=1}^\infty \dfrac{1}{(n + 2)(n + 3)} = \dsum_{n=1}^\infty \dfrac{1}{(n + 2)}- \dsum_{n=1}^\infty \dfrac{1}{ (n + 3)} !$$


(Oral CCP)

Soit $(u_n)$ une suite à termes positifs telle que $\dsum u_n$ diverge. Donner la nature et la somme de $\dsum\dfrac{u_n}{(1+u_0)\cdots(1+u_n)}$.

Correction

Il suffit de remarquer que, pour $n\geq 1$, $$\dfrac{u_n}{(1+u_0)\cdots(1+u_n)}=\dfrac{1}{(1+u_0)\cdots(1+u_{n-1})}-\dfrac{1}{(1+u_0)\cdots(1+u_n)}. $$ Ensuite, remarquer que, $$\forall n\in \N,\quad (1+u_0)\cdots(1+u_n)\geq \dsum_{k=0}^n u_k \tendversN\,\infty \Longrightarrow \dfrac{1}{(1+u_0)\cdots(1+u_n)} \tendversN \,0.$$


Séries à termes positifs


Rien

Rapport du jury -- CCP 2014
Certains éprouvent des difficultés à montrer la convergence d'une série ou d'une intégrale sur des exemples simples (et classiques). L'utilisation des théorèmes de comparaison, pour les séries ou les intégrales impropres, sans se soucier des questions de signe est sanctionnée.


Règles de comparaison

Une série à termes réels positifs est convergente ssi la suite de ses sommes partielles est majorée.

Soient $(u_n)$ et $(v_n)$ deux suites de nombres réels telles que $$0 \leq u_n \leq v_n \quad \text{à partir d'un certain rang}.$$ Alors:

  • Si la série $\sum v_n$ converge, la série $\sum u_n$ converge.
  • Si la série $\sum u_n$ diverge, la série $\sum v_n$ diverge.

Exemples

  1. Étude de la série $\dsum \dfrac1{n^2}$ :
    On remarque que, pour $n\geq 2$, $0 \leq \dfrac1{n^2} \leq \dfrac1{n(n-1)}$. Or $\dfrac1{n(n-1)} = \dfrac1{n-1}-\dfrac1n$, donc la série $\dsum \dfrac1{n(n-1)}$ converge.
    D'après le théorème précédent : La série $\dsp\sum_{n\in\N^*}\dfrac1{n^2}$ converge.
  2. Il résulte alors du même théorème que : $\alpha \geq 2 \Longrightarrow \dsp\sum \dfrac1{n^{\alpha}}$ converge.
  3. Puisque la série harmonique $\dsum \dfrac1n$ diverge, on obtient aussi : $\alpha \leq 1 \Longrightarrow \dsp\sum \dfrac1{n^{\alpha}}$ diverge.

(Une partie d'orale X 2016)

Soient $(a_n)$ et $(c_n)$ deux suites à termes strictement positifs. On suppose que $\dsum \dfrac{1}{c_n}$ diverge.

Pour $n\in \N$, on pose $k_n=c_n\dfrac{a_n}{a_{n+1}}-c_{n+1}.$

  1. On suppose qu'il existe $n_0\geq 0$ et $\alpha>0$ tels que : $\forall n\geq n_0,\quad k_n\geq \alpha$. Montrer alors que $\dsum a_n $ converge.
  2. On suppose qu'il existe $n_0\geq 0$ tel que : $\forall n\geq n_0,\quad k_n\leq 0$. Montrer alors que $\dsum a_n $ diverge.
Correction

  1. Soit $n\geq n_0+1$, on a: $$a_{n}k_{n-1}=c_{n-1}a_{n-1}-a_{n}c_{n}> a_{n}\alpha >0$$ Ce qui donne $$\forall n\geq n_0+1,\quad c_{n_0}a_{n_0}\geq c_{n_0}a_{n_0}-c_{n}a_{n}=\dsum_{k=n_0+1}^n c_{k-1}a_{k-1}-c_ka_k>\dsum_{k=n_0+1}^n\alpha a_k>0$$ ce qui implique que $\dsum a_k $ converge.
  2. Soit $n\geq n_0+1$, on a: $$a_{n}k_{n-1}=c_{n-1}a_{n-1}-a_{n}c_{n}\leq 0 \Longrightarrow (a_nc_n)_{n\geq n_0+1} \text{ est croissante}$$ Ce qui donne $$\forall n\geq n_0+1,\quad a_nc_n\geq a_{n_0}c_{n_0}\Longrightarrow a_n\geq \dfrac{a_{n_0}c_{n_0}}{c_n} $$ Vu que $\dsum \frac{1}{c_n}$ diverge, il en va de même pour $\dsum a_k $.

Soient $(u_n)$ et $(v_n)$ deux suites de nombres réels positifs. On suppose que : $u_n =\underset{n\to\infty}{\mathrm{O}}(v_n)$.
Alors, si la série $\sum v_n$ converge, la série $\sum u_n$ converge.

Soient $(u_n)$ et $(v_n)$ deux suites de nombres réels positifs. On suppose que : $u_n =\underset{n\to\infty}{\mathrm{o}}(v_n)$.
Alors, si la série $\sum v_n$ converge, la série $\sum u_n$ converge.

Soit $(u_n)$ et $(v_n)$ deux suites de nombres réels positifs. On suppose que: $u_n \underset{\infty}{\thicksim}v_n$.
Alors : les séries $\sum u_n$ et $\sum v_n$ sont de même nature.

Exemples

  1. $\dsum \left(\sqrt{1+n+n^2}-n-\frac{1}{2}\right) $ est divergente, en effet, $$\begin{array}{lcl} \sqrt{1+n+n^2}-n-\frac{1}{2}&=&n\left(1+\frac{1}{n}+\frac{1}{n^2}\right)^{\frac{1}{2}}-n-\frac{1}{2}\\ &=&n\left(1+\dfrac{1}{2n}+\dfrac{3}{8n^2}+\mathrm{O}\left(\frac{1}{n^3}\right)\right)-n-\frac{1}{2}\\ &=&\dfrac{3}{8n}+\mathrm{O}\left(\frac{1}{n^2}\right)\underset{n\to\infty}{\thicksim}\dfrac{3}{8n} \end{array}$$
  2. $\dsum \dsp n+\dfrac12-\dfrac1{\ln(n^2+n+1)-\ln(n^2+1)}$, diverge, en effet, en faisant un DL on trouve $$\begin{array}{lcl} \ln(n^2+n+1)-\ln(n^2+1)&=&\ln\left(1+\dfrac{1}{n}+\dfrac{1}{n^2}\right)-\ln\left(1+\dfrac{1}{n^2}\right)\\ &&\\ & =&\dfrac{1}{n}+\dfrac{1}{n^2}-\dfrac{1}{2}\left(\dfrac{1}{n^2}+\dfrac{1}{n^3}\right)+\dfrac{1}{3n^3}-\dfrac{1}{n^2}+\underset{n\to\infty}{\mathrm{O}}\left(\dfrac{1}{n^4}\right)\\ &&\\ &=&\dfrac{1}{n}-\dfrac{1}{2n^2}-\dfrac{2}{3n^3}+\underset{n\to\infty}{\mathrm{O}}\left(\dfrac{1}{n^4}\right) \end{array}$$ ce qui donne, $$\begin{array}{lcl} n+\dfrac{1}{2}-\dfrac{n}{1-\dfrac{1}{2n}-\dfrac{2}{3n^2}+ \underset{n\to\infty}{\mathrm{O}}\left(\dfrac{1}{n^3}\right)}&=& n+\dfrac{1}{2}-n\left(1+\dfrac{1}{2n}+\dfrac{2}{3n^2}+\dfrac{1}{4n^2} + \underset{n\to\infty}{\mathrm{O}}\left(\dfrac{1}{n^3}\right)\right)\\ &=&\dfrac{-11}{12n}+\underset{n\to\infty}{\mathrm{O}}\left(\dfrac{1}{n^2}\right) \end{array}$$

Danger

On ne peut RIEN dire si $u_n \thicksim v_n$ et si $(u_n), (v_n)$ ne sont pas de signes constants.

Rien

Rapport du jury -- CCP 2017 Orale
Pour les théorèmes de comparaison des séries à termes positifs ou des intégrales généralisées, beaucoup de candidats les appliquent sans justement vérier le signe des termes et éventuellement évoquer l'absolue convergence. La manipulation des équivalents est souvent hasardeuse.

Exemple

$u_n=\dfrac{(-1)^n}{\sqrt{n}}$ et $v_n=\dfrac{(-1)^n}{\sqrt{n}} + \dfrac1n$. On a $u_n \underset{\infty}{\thicksim} v_n$ , pourtant $\dsum u_n $ converge (voir section séries alternées, théorème CSSA )et $\dsum v_n$ diverge puisque $\dsum \dfrac{1}{n}$ diverge.


Soit $(u_n)_n$ une suite de réels positifs. On pose $v_n=\dfrac{u_n}{1+u_n}$. Montrer que $\dsum u_n$ et $\dsum v_n$ sont de même nature.

Correction

On distingue deux cas:

  1. Si $u_n\tendversN\,0$ alors $u_n\underset{\infty}{\thicksim}v_n$ puisque $\dfrac{v_n}{u_n}=\dfrac{1}{1+u_n}\tendversN\,1$, en utilisant le résultat du théorème précédent, on trouve $\dsum u_n$ et $\dsum v_n$ sont de même nature.
  2. Si $(u_n)$ ne tend pas vers $0$ (donc $\dsum u_n$ diverge), alors $(v_n)$ ne tend pas vers $0$, en effet: $$\forall \varepsilon>0,\,\, \forall n\in \N,\,\exists m\geq n, \,\,u_m>\varepsilon, \text{or } v_m=\dfrac{u_m}{1+u_m}\geq \dfrac{\varepsilon}{1+\varepsilon}>0.$$ (La fonction $x\longmapsto \dfrac{x}{1+x}$ est croissante sur $\R_+$), donc $\dsum v_n$ diverge, i.e. $\dsum u_n$ et $\dsum v_n$ sont de même nature.

Méthode d'études

On appelle série de Riemann toute série de la forme $\dsp\sum_{n\geq 1} \dfrac{1}{n^\alpha}$ où $\alpha\in \R$.

Le série de Riemann $\dsp\sum_{n\geq 1} \dfrac{1}{n^\alpha}$ converge si et seulement si $\alpha>1$.

Remarque

On peut aussi retrouver ce résultat en utilisant la comparison sérié-intégrale .

Soient $\sum u_n$ une série à termes réels strictement positifs.

  • On suppose qu'il existe $k>0$ et $\alpha\in \R$ tels que $u_n\thicksim \dfrac{k}{n^\alpha}$.
    1. Si $\alpha >1$, la série $\dsp\sum u_n $ converge.
    2. Si $\alpha \leq 1$, la série $\dsp\sum u_n $ diverge.
  • S'il existe $\alpha>1$ tel que $u_n=\underset{n\to\infty}{\mathrm{O}} \left(\dfrac{1}{n^\alpha}\right)$, alors la série $\dsp\sum u_n $ converge.
  • S'il existe $\alpha\leq 1$ et $\lambda>0$ tels que $u_n\geq \dfrac{\lambda}{n^\alpha}$ (a partir d'un certain rang), alors la série $\dsp\sum u_n $ diverge.
Remarque

On trouve aussi un résultat similaire pour l'étude des intégrales impropres .

Soient $\sum u_n$ et $\sum v_n$ deux séries à termes réels strictement positifs.
On suppose : $\exists n_0\in \N,\ \forall n\geq n_0, \ \dfrac{u_{n+1}}{u_n} \leq \dfrac{v_{n+1}}{v_n}$. Alors :

  • Si la série $\sum v_n$ converge, la série $\sum u_n$ converge.
  • Si la série $\sum u_n$ diverge, la série $\sum v_n$ diverge.

(Théorème de D'Almbert) Soit $\sum u_n$ une série à termes réels strictement positifs.

  1. S'il existe $k \in ]0,1[$ tel que $\dfrac{u_{n+1}}{u_n}\leq k$ (à partir d'un certain rang), alors la série $\sum u_n$ converge.
  2. Si $\dfrac{u_{n+1}}{u_n} \geq 1$ à partir d'un certain rang, la série $\sum u_n$ diverge.

(Règle de D'Almbert) Soit $\sum u_n$ une série à termes réels strictement positifs, telle que $\dsp\lim_{n\to+\infty}\left(\dfrac{u_{n+1}}{u_n}\right)=\ell$ existe dans $\overline{\R}$ ($\ell \in [0,+\infty]$).

  1. Si $\ell < 1$, la série $\sum u_n$ converge.
  2. Si $\ell>1$, la série $\sum u_n$ diverge.
Remarque

Si $\,\ell=1$,on ne peut rien dire a priori.

(d'après orale Centrale PSI)

On donne $u_0=1,\,\dfrac{u_{n+1}}{u_n}=\dfrac{n+a}{n+b}$ où $a,\,b\in \R_+^*$, on pose $v_n=\ln\left(n^{b-a}u_n\right)$.
Montrer la convergence de la série $\dsum (v_{n+1}-v_n)$ et en déduire une condition sur $a$ et $b$ pour que $\dsum u_n$ converge.

Correction

On vérifie facilement que $u_n>0$ pour tout $n\in\N$, donc $v_n$ est bien définie.D'autre part, $$\begin{array}{lcl} v_{n+1}-v_n&=&\dsp\ln\left((n+1)^{b-a}u_{n+1}\right)-\ln\left(n^{b-a}u_n\right)=\ln\left(\left(1+\frac{1}{n}\right)^{b-a}\frac{u_{n+1}}{u_n}\right)\\ &&\\ &=&\dsp\ln\left(\left[1+\frac{b-a}{n}+\frac{(b-a)(b-a-1)}{n^2}+\underset{\infty}{\mathrm{o}}\left(\frac{1}{n^2}\right)\right]\left(1+\frac{a}{n}\right)\left[1-\frac{b}{n}+\frac{b^2}{n^2}+\underset{\infty}{\mathrm{o}}\left(\frac{1}{n^2}\right)\right]\right)\\ &&\\ &=&\dsp \ln\left(1+\dfrac{(b-a)(b+a-1)}{2n^2}++\underset{n\to\infty}{\mathrm{o}}\left(\frac{1}{n^2}\right)\right) \\ &&\\ &=&\dsp \dfrac{(b-a)(b+a-1)}{2n^2}++\underset{n\to\infty}{\mathrm{o}}\left(\frac{1}{n^2}\right) \end{array} $$ On en déduit que la série $\dsum v_{n+1}-v_n$ converge,ce qui implique que la suite $(v_n)$ tend vers une limite fini $\ell$. On en déduit alors, $$n^{b-a}u_n\tendversN\,\ee^\ell >0 \Longrightarrow u_n\underset{n\to\infty}{\sim}\dfrac{\ee^\ell}{n^{b-a}}$$ Conclusion: $$\boxed{ \dsum u_n \text{ converge ssi }b-a>1.}$$

Remarque

En effet, on a pour tout $n\geq 1$, $$ (n+b)u_{n+1}=(n+a)u_n\Longrightarrow (n+1)u_{n+1}-nu_n=au_n-(b-1)u_{n+1} $$ En additionnant cette relation pour $n\in \inter{0,N}$ (avec $N\geq 1$), on trouve, $$ \begin{array}{lcl} (N+1)u_{N+1}&=&aS_{N}-(b-1)(S_{N+1}-u_0)\\ &=&(a-b+1)S_N-(b-1)U_{N+1}+(b-1)u_0 \end{array} $$ Autrement dit, pour tout $ n\geq 1$, $$(n+b)u_{n+1}=(a-b+1)S_n+(b-1)u_0=(a-b+1)\dsum_{k=1}^nu_k+au_0\quad(\star)$$ On distingue différents cas

  1. Si $(a-b+1)\geq 0$ alors d'après $(\star)$, $(n+b)u_{n+1}=(n+a)u_n\geq a u_0$ donc $\dsum u_k$ diverge.
  2. Si $(a-b+1) < 0$ alors $S_n\leq \dfrac{a u_0}{b-1-a}$ ( d'après la relation $(\star)$ puisque $u_n>0$) donc la suite $(S_n)$ est majorée donc $\dsum u_n$ converge.
    Notons alors $\ell$ la somme de cette série, si $(a-b+1)\ell+a u_0\neq 0$ alors $u_n\underset{n\to\infty}{\sim }\dfrac{(a-b+1)\ell+a u_0}{n+a}$ ce qui contredit la convergence de $\dsum u_n$.
    Conclusion $$\dsum_{k=0}^\infty u_k = \ell =\dfrac{ b-1}{b-1-a}u_0$$

Séries alternées

Une série à termes réels $\sum u_n$ est dite alternée si la suite $((-1)^nu_n)$ est de signe constant.

Soit $\sum u_n$ une série alternée. On suppose que

  1. la suite $(\abs{u_n})$ est décroissante;
  2. $\dsp\lim_{n\to\infty}u_n=0$.

Alors la série $\sum u_n$ converge.

Exemples

  1. Les séries de Riemann alternées $\dsum_{n\in \N^*} \dfrac{(-1)^{n-1}}{n^{\alpha}}$
    Posons, pour $n\geq 1$, $u_n=\dfrac{(-1)^{n-1}}{n^{\alpha}}$.
    1. Si $\alpha\leq0$, la suite $(u_n)$ ne tend pas vers $0$ quand $n\to\infty$, donc la série $\sum u_n$ diverge grossièrement.
    2. Si $\alpha>1$, $\abs{u_n}=\dfrac1{n^{\alpha}}$ et la série $\sum u_n$ est absolument convergente, donc convergente.
    3. Si $\alpha\in]0,1[$, la suite $(u_n)$ vérifie les hypothèses du CSSA, donc la série$\sum u_n$ converge (elle est ici semi-convergente).
  2. Étude de la série de terme général $u_n=(-1)^n\sqrt{n}\sin\left(\dfrac1n\right)$ ($n\geq 1)$.
    On effectue un DL de $u_n$: $u_n=(-1)^n\sqrt{n}\left( \dfrac1n+\underset{n\to\infty}{\mathrm{O}}\left(\dfrac{1}{n^3}\right)\right)=\dfrac{(-1)^n}{\sqrt{n}}+ \underset{n\to\infty}{\mathrm{O}}\left(\dfrac{1}{n^{\frac52}}\right)$. Ainsi, $u_n=v_n+w_n$, avec $v_n=\dfrac{(-1)^n}{\sqrt{n}}$ et $w_n=\underset{n\to\infty}{\mathrm{O}}\left(\dfrac{1}{n^{\frac52}}\right)$. La série de terme général $v_n$ est convergente (série de Riemann alternée), et celle de terme général $w_n$ est absolument convergente (comparaison à une série de Riemann). Il en résulte que $\sum u_n$ est la somme de deux séries convergentes, donc est convergente.


On considère la suite $(u_n)_{n\geq 1}$ définie par $u_n=\dfrac{1}{n}$ si $n$ est paire, $\dfrac{-1}{n^2}$ sinon.
Étudier la convergence de $\dsum u_n$.

Correction

On a $(-1)^nu_n>0$ et $u_n\tendversN\,0$ puisque $\abs{u_n}\leq \dfrac{1}{n}$.
Soit $n>0$, on a: $$S_{2n}=\dsum_{k=1}^{2n}u_k=\dsum_{k=0}^{n-1}u_{2k+1}+\dsum_{k=1}^nu_{2k}= -\dsum_{k=0}^{n-1}\dfrac{1}{(2k+1)^2}+\dsum_{k=1}^n\dfrac{1}{2k}=-V_n+\dfrac{1}{2}H_n$$ On sait déjà que $H_n\tendversN\,\infty$, d'autre part, $$\forall n\geq 2,\quad V_n\leq 1+\dsum_{k=1}^n\dfrac{1}{4k^2}\leq 1+ \dfrac{1}{4}\dsum_{k\in \N^*}\dfrac{1}{k^2}=1+\dfrac{\pi^2}{6}$$ Ce qui implique que la suite $(V_n)$ est bornée. On en déduit alors que $S_{2n}\tendversN\,\infty$ ce qui implique $\dsum u_n $ diverge.

Soit $\sum u_n$ une série alternée telle que

  1. la suite $(\abs{u_n})$ est décroissante;
  2. $\dsp\lim_{n\to\infty}u_n=0$.

et soit $U$ sa somme. Alors:

  • $U$ est comprise entre deux sommes partielles d'indices consécutifs.
  • $U$ est du signe de $u_0$, et $\abs{U}\leq \abs{u_0}$.
  • Si on note $R_n=\dsum_{k=n+1}^{\infty}$ le reste d'ordre $n$, alors $R_n$ est du signe de $u_{n+1}$ et $\abs{R_n}\leq \abs{u_{n+1}}$

Rien

Rapport du jury -- Mines 2015
Le critère des séries alternées est connu mais la majoration des restes l’est moins (décalage d’indice, oubli de la valeur absolue...). De façon générale, l’étude des séries semi-convergentes qui ne vérifient pas ce critère est mal faite. En particulier, l’utilisation d’un développement limité n’est pas un réflexe.

Exemple

Ci-après une animation montrant le comportement de $S_n$ par rapport à sa milite.


Soit $x\in [0,1]$, on note $S(x)=\dsum_{n=0}^\infty\dfrac{(-1)^n x^{2n+1}}{2n+1}$.

  1. Montrer que $S(x)$ converge et que $S(x)=\Arctan(x)$.
  2. Justifier la relation suivante: \begin{equation} %\label{Eq1} \pi=4\dsum_{k=0}^\infty \dfrac{(-1)^{k}}{2k+1} \end{equation} Pour $n\in \N^*$. Donner une majoration de $\abs{\pi -4 \dsum_{k=0}^n \dfrac{(-1)^{k}}{2k+1}}$, en déduire la plus petite valeur de $n$ pour qu'on ait $\dsp\abs{\pi -4 \dsum_{k=0}^n \dfrac{(-1)^{k}}{2k+1}}\leq 10^{-6}.$
  3. Justifier la relation suivante: \begin{equation} %\label{Eq2} \pi=4\dsum_{k=0}^\infty \dfrac{(-1)^{k}}{2k+1}\left(\frac{1}{2^{2k+1}}+\frac{1}{3^{2k+1}}\right) \end{equation} Pour $n\in \N^*$. Donner une majoration de $\abs{\pi -4 \dsum_{k=0}^n \dfrac{(-1)^{k}}{2k+1}\left(\frac{1}{2^{2k+1}}+\frac{1}{3^{2k+1}}\right)}$, en déduire la plus petite valeur de $n$ pour qu'on ait $\dsp\left|\pi -4 \dsum_{k=0}^n \dfrac{(-1)^{k}}{2k+1}\left(\frac{1}{2^{2k+1}}+\frac{1}{3^{2k+1}}\right)\right|\leq 10^{-6}.$

Correction

  1. Soit $x\in [0,1]$ la série $S(x)$ vérifie les conditions de théorème précédent, donc $S(x) $ converge. ( Pour $x\in [0,1[$, $S(x)$ est absolument convergente). Soit $n\geq 1$, on a $$\begin{array}{lcl} S_n(x)&=&\dsum_{k=0}^n\dfrac{(-1)^kx^{2k+1}}{2k+1}=\dsum_{k=0}^n(-1)^k\int_0^xt^{2k}\ud t =\dsp \int_0^x\left(\sum_{k=0}^n(-t^2)^{k}\right)\ud t\\ &&\\ &=&\dsp\int_0^x\dfrac{1-(-t^2)^{n+1}}{1+t^2}\ud t=\dsp\int_0^x\dfrac{1}{1+t^2}\ud t-\int_0^x\dfrac{(-t^2)^{n+1}}{1+t^2}\ud t\\ &&\\ &=&\Arctan(x)+\dsp\int_0^x\dfrac{(-t^2)^n}{1+t^2}\ud t \end{array}$$ On en déduit alors, $$\abs{S_n(x)-\Arctan(x)}\leq \abs{\int_0^x\dfrac{(-t^2)^{n+1}}{1+t^2}\ud t}\leq \int_0^xt^{2n+2}\ud t=\dfrac{x^{2n+3}}{2n+3}\tendversN\,0$$ Ce qui prouve que $S(x)$ est convergente, de plus, $$\boxed{\forall x\in [0,1],\quad \dsum_{n\geq 0}\dfrac{(-1)^nx^{2n+1}}{2n+1} =\Arctan(x)}.$$
  2. En particulier, pour $x=1$, on trouve $\dsum_{n\geq 0}\dfrac{(-1)^n}{2n+1} =\Arctan(1)=\dfrac{\pi}{4}$.
    En utilisant le théorème du majoration du reste pour les séries alternées, on trouve: $$\forall n\geq 0,\quad\abs{\dfrac{\pi}{4}-\dsum_{k=0}^n\dfrac{(-1)^n}{2k+1}}\leq \dfrac{1}{2n+3}\Longrightarrow \abs{\pi -4 \dsum_{k=0}^n \dfrac{(-1)^{k}}{2k+1}}\leq\dfrac{4}{2n+3}$$ Donc si on souhaite une approximation de $\pi$ en utilisant cette relation à $10^{-6}$ il faut prendre $n\geq n_0=1999999$.
  3. Pour montrer la deuxième relation, il suffit de remarquer que $$\Arctan (1/2)+\Arctan(1/3)=\Arctan\left(\dfrac{\frac{1}{2}+\frac{1}{3}}{1-\frac{1}{2}\frac{1}{3}}\right)=\Arctan\left(\dfrac{\frac{5}{6}}{1-\frac{1}{6}}\right) =\Arctan (1)=\dfrac{\pi}{4},$$ puisque $1/2,\,1/3\in [0,1]$, on utilise alors la fonction $S$, ce qui donne, $$\dfrac{\pi}{4}=\dsum_{k\geq 0}\dfrac{(-1)^k}{(2k+1)2^{2k+1}}+\dsum_{k\geq 0}\dfrac{(-1)^k}{(2k+1)3^{2k+1}}\Longrightarrow \pi= 4\dsum_{k=0}^\infty \dfrac{(-1)^{k}}{2k+1}\left(\frac{1}{2^{2k+1}}+\frac{1}{3^{2k+1}}\right).$$ En utilisant le théorème du majoration du reste pour les séries alternées, on trouve: $$\abs{\pi -4 \dsum_{k=0}^n \dfrac{(-1)^{k}}{2k+1}\left(\frac{1}{2^{2k+1}}+\frac{1}{3^{2k+1}}\right)}\leq \dfrac{4}{2n+3}\left(\dfrac{1}{2^{2n+3}}+\dfrac{1}{3^{2n+3}}\right).$$ Pour obtenir une erreur inférieur à $10^{-6}$ il faut prendre $n\geq n_1=8$.


  1. Montrer que la série $\dsum_{n\geq 1}\dfrac{(-1)^n}{n}$ converge, et donner une majoration de $R_n$ ($n\in \N$).
  2. A l'aide d'une suit géométrique, montrer que, pour tout $n\geq 0$, $\dsp R_n=(-1)^{n+1}\int_0^1\dfrac{x^n\ud x}{1+x}$.
  3. Par une IPP, montrer que $R_n=K\dfrac{(-1)^{n+1}}{n^{\beta}}+\underset{n\to\infty}{\mathrm{O}}\left(\frac{1}{n^{\beta+1}}\right)$ avec $K\in \R^*,\,\beta\in \N^*$.

En déduire la nature de la série de terme général $R_n$, puis calculer sa somme.

Correction

  1. La série $\dsum\dfrac{(-1)^n}{n}$ est alternée. De plus la suite $(\frac{1}{n})_{n\geq 1}$ est strictement décroissante et tend vers $0$. Donc d'après le CSSA, la série $\dsum\dfrac{(-1)^n}{n}$ converge, de plus, on a $$\forall n\in \N^*,\quad \abs{R_n}\leq \dfrac{1}{n+1}.$$
  2. Soient $n\in \N,\,m\in \N$ tel que $m\geq n$, pour tout $x\in [0,1]$, on a: $$\dfrac{x^n}{1+x}=\dfrac{x^n}{1-(-x)}=x^n\dsum_{k=0}^m(-x)^k+x^n\dfrac{(-x)^{m+1}}{1+x}=(-1)^n\dsum_{k=n}^{n+m}(-1)^kx^k-\dfrac{(-1)^{m}x^{n+m}}{1+x}.$$ En intégrant la relation précédente entre $0$ et $1$, on trouve: $$\begin{array}{lcl} (-1)^{n+1}\int_0^1\dfrac{x^n\ud x}{1+x}&=&-\dsum_{k=n}^{n+m}\int_0^1(-x)^k\ud x-\int_0^1\dfrac{(-1)^{m}x^{n+m}}{1+x}\ud x\\ &=&\dsum_{k=n}^{n+m}\dfrac{(-1)^{k+1}}{k+1}-\int_0^1\dfrac{(-1)^{m}x^{n+m}}{1+x}\ud x. \end{array}$$ Ainsi, pour tout $m\geq n$, on a $$\abs{(-1)^{n+1}\int_0^1\dfrac{x^n\ud x}{1+x}-\dsum_{k=n+1}^{n+m+1}\dfrac{(-1)^{k}}{k}}\leq \abs{\int_0^1\dfrac{(-1)^{m}x^{n+m}}{1+x}\ud x}\leq \int_0^1x^{n+m}\ud x\tendvers{m}{\infty}0$$ Donc $\left(\dsum_{k=n+1}^{m}\frac{(-1)^k}{k}\right)_{m\geq n+1}$ converge vers $(-1)^{n+1}\dsp\int_0^1\dfrac{x^n\ud x}{1+x}$, d'autre part cette suite converge également vers $R_n$, donc par unicité de la limite, on trouve $$\boxed{ R_n=(-1)^{n+1}\int_0^1\dfrac{x^n\ud x}{1+x}}.$$
  3. Soit $n\in \N^*$, on a $$(-1)^{n+1}R_n=\int_0^1\dfrac{x^n\ud x}{1+x}=\int_0^1x^{n-1}\dfrac{x}{1+x}\ud x=\left[\dfrac{x^n}{n}\dfrac{x}{1+x}\right]_0^1-\int_0^1\dfrac{x^n}{n}\dfrac{1}{(1+x)^2}\ud x=\dfrac{1}{2n}+I_n.$$ Avec $$\abs{I_n}=\int_0^1\dfrac{x^n}{n(1+x)^2}\ud x\leq \dfrac{1}{n}\int_0^1x^n\ud x=\dfrac{1}{2n(n+1)}\Longrightarrow I_n=\underset{n\to\infty}{\mathrm{O}}\left(\frac{1}{n^2}\right).$$ On en déduit $\boxed{R_n=\dfrac{(-1)^{n+1}}{2n}+ \underset{n\to\infty}{\mathrm{O}}\left(\frac{1}{n^2}\right)}$. Donc on peut prendre $\beta =1$ et $K=\dfrac{1}{2}$.
  4. D'après la question précédente, on a pour $n\geq 1$, $R_n=\dfrac{(-1)^{n+1}}{2n}+w_n$ avec $w_n=\underset{n\to\infty}{\mathrm{O}}\left(\frac{1}{n^2}\right)$, donc $R_n$ est la somme de deux termes généraux des séries convergentes, donc $\dsum R_n$ converge.
    Soit $n\geq 1$, on a $$\begin{array}{lcl} \dsum_{k=0}^nR_k&=&\dsum_{k=0}^n(-1)^{k+1}\int_0^1\dfrac{x^k\ud x}{1+x}=-\int_0^1\left(\dsum_{k=0}^n(-x)^k\right)\dfrac{\ud x}{1+x}=-\int_0^1\dfrac{1-(-x)^{n+1}}{1+x}\dfrac{\ud x}{1+x}\\ &&\\ &=&-\int_0^1\dfrac{\ud x}{(1+x)^2}+(-1)^{n+1}\int_0^1\dfrac{x^{n+1}\ud x}{(1+x)^2}\tendversN-\int_0^1\dfrac{\ud x}{(1+x)^2} \end{array}$$ On en déduit alors $\boxed{\dsum_{n\geq 0}R_n=-\left[\dfrac{-1}{(1+x)}\right]_0^1=\dfrac{-1}{2}}$.

Comparaison série-intégrale

Soit $f$ une fonction continue par morceaux sur un intervalle de la forme $[n_0,+\infty[$ ($n_0\in\N$), à valeurs réelles positives et décroissante.
Alors la série de terme général $ w_n=\dsp\int_n^{n+1} f(t)\ud t-f(n+1) \ (n \geq n_0)$ est convergente.

Illustration graphique

Soit $f$ une fonction continue par morceaux sur un intervalle de la forme $[n_0,+\infty[$ ($n_0\in\N$), à valeurs réelles positives et décroissante. Alors : $$\text{la série }\sum f(n) \text{ converge ssi } \dsp\int_{n_0}^{+\infty}f \text{ existe}.$$

Remarque

Il est tout aussi important de retenir la démonstration que le résultat de ces théorèmes. En effet, la méthode de comparaison série-intégrale permet d'obtenir facilement un encadrement des sommes partielles (ou du reste, en cas de convergence).

Soit $f$ une fonction continue par morceaux sur un intervalle de la forme $[n_0,+\infty[$ ($n_0\in\N$), à valeurs réelles positives et décroissante. Posons $u_n=f(n)$ pour $n\geqslant n_0$. Alors: $$\exists \ell \in\R \text{ tq } \dsp\sum_{k=n_0}^n u_k = \dsp\int_{n_0}^n f(t) \ud t + \ell +\underset{n\to\infty}{\mathrm{o}}(1).$$

Exemple

Prenons $f(t)=\dfrac1t$ pour $t\geq 1$. Cette fonction vérifie bien les hypothèses du théorème (avec ici $n_0=1$), donc le résultat précédent s'écrit: $$\exists \gamma \in\R \text{ tq } \dsp\sum_{k=1}^n \dfrac1k = \ln n + \gamma +\underset{n\to\infty}{\mathrm{o}}(1).$$ $\gamma$ s'appelle la constante d'Euler ; on a $\gamma\approx0,57721...$


On considère la série $\dsum \dfrac{1}{n^3}$ (une série convergente), on note $S$ sa somme, et pour tout $n\geq 1$, on note $$S_n=\dsum_{k=1}^n\dfrac{1}{k^3}\quad \text{et}\quad R_n=\dsum_{k\geq n+1}\dfrac{1}{k^3}.$$

  1. A l'aide d'une comparaison avec une intégrale, montrer que: $$\forall n\geq 1,\quad \dsp\dfrac{1}{2(n+1)^2}\leq R_n\leq \dfrac{1}{2n^2}.$$ En déduire le plus petit entier $n$ tel que $\abs{S-S_n}\leq 10^{-6}$.
  2. On pose, pour $n\geq 1$, $A_n=S_n+\dfrac{1}{2}\left(\dfrac{1}{2(n+1)^2}+\dfrac{1}{2n^2}\right)$. En utilisant la question précédente, montrer que: $$\forall n\in \N^*,\quad \abs{A_n-S}\leq \dfrac{1}{2n^3}.$$ En déduire le plus petit entier $n$ tel que $\abs{S-A_n}\leq 10^{-6}$.

Correction

  1. La fonction $f:x\longmapsto \dfrac{1}{x^3}$ est strictement décroissante sur $\R_+^*$. $$\forall k\geq 2,\, \int_k^{k+1}f(x)\ud x\leq \dfrac{1}{k^3}\leq \int_{k-1}^kf(x)\ud x$$ Soient $ N>n\geq 1$, on a: $$ \int_{n+1}^{N+1}f(x)\ud x\leq \dsum_{k=n+1}^N\dfrac{1}{k^3}\leq \int_n^N f(x)\ud x\Longrightarrow \dfrac{1}{2(n+1)^2}-\dfrac{1}{2(N+1)^2}\leq \dsum_{k=n+1}^N\dfrac{1}{k^3}\leq \dfrac{1}{2n^2}-\dfrac{1}{2N^2}.$$ En faisant $N$ tend vers $\infty$, on trouve $$\forall n\geq 1,\quad \dsp\dfrac{1}{2(n+1)^2}\leq R_n\leq \dfrac{1}{2n^2}.$$ Ainsi, si on souhaite que $R_n$ soit inférieur à $10^{-6}$ il suffit de prendre $n$ tel que $\dfrac{1}{2n^2}\leq 10^{-6}$ soit $n\geq 708$.
  2. Soit $n\geq 1$, on a $$\begin{array}{lcl} \abs{S-A_n}&=&\abs{S_n+R_{n}-\left(S_n+\dfrac{1}{2}\left(\dfrac{1}{2(n+1)^2}+\dfrac{1}{2n^2}\right)\right)}=\abs{R_n-\dfrac{1}{2}\left(\dfrac{1}{2(n+1)^2}+\dfrac{1}{2n^2}\right)}\\ &&\\ &\leq &\dfrac{1}{2}\left(+\dfrac{1}{2n^2}-\dfrac{1}{2(n+1)^2}\right)=\dfrac{1}{4}\dfrac{2n+1}{n^2(n+1)^2}\leq \dfrac{1}{2n^3} \end{array}$$ Ainsi, si on souhaite que $\abs{S-A_n}$ soit inférieur à $10^{-6}$ il suffit de prendre $n$ tel que $\dfrac{1}{2n^3}\leq 10^{-6}$ soit $n\geq 80$.

Conclusion, pour avoir une valeur approchée de $S$ à $10^{-6}$ près, il suffit de calculer $S_{80}$ puis ajouter $\dfrac{1}{4}\left(\dfrac{1}{6400}+\dfrac{1}{6561}\right)$.
En utilisant le logiciel {\tt{Maple}}, on trouve les résultats suivants: $$\begin{array}{lcl} S &=&1.202\,\,056\,\,903\,\,159\,\,594\,\,285\,\,399\,\,\cdots\\ S_{708}&=&1.202\,\,055\,\,907\,\,089\,\,092\,\,414\,\,947\,\,\cdots \\ S_{80} &=&1.201\,\,979\,\,748\,\,618\,\,896\,\,502\,\,181\,\,\cdots\\ A_{80} &=&1.202\,\,056\,\,915\,\,066\,\,465\,\,470\,\,327\,\,\cdots\\ R_{708}&=& .9960705018704517858608418878894904215906704e-6\\ \abs{S-A_{80}}&=&.119068711849273283819093965990942375626486e-7 \end{array}$$

Remarque

On peut encore accélérer la convergence en utilisant d'autre méthodes. Par exemple, si on pose $$V_n=S_n +\dfrac{1}{2n^2}-\dfrac{1}{2n^3}+\dfrac{1}{4n^4},$$ on peut montrer que $\abs{S-V_n}\leq \dfrac{1}{12(n-1)^6}$. Donc si on calcul $S_8$ et on corriger en ajoutant $\dfrac{1}{128}-\dfrac{1}{1024}+\dfrac{1}{16384}$ à $S_8$ on obtient une approximation de $S$ à $10^{-6}$.

Séries absolument convergentes

Définitions

Une série $\dsum u_n$ d'éléments de $\C$ est dite absolument convergente si la série (à termes réels positifs) $\dsum |u_n|$ est convergente.

Soit $\sum z_n$ une série de terme général $z_n\in\C$, avec pour tout $n$, $z_n=a_n+\ii b_n$, $a_n,b_n \in\R$.
Alors : $\sum z_n$ est absolument convergentes ssi les deux séries $\sum a_n$ et $\sum b_n$ sont absolument convergentes.

Remarque

Il existe des séries qui sont convergentes, mais pas absolument convergentes. Par exemple, la série harmonique alternée. Une telle série est dite semi-convergente.

Toute série absolument convergente de nombres complexes est convergente.

Les espaces vectoriels $\ell^1(\C)$ et $\ell^2(\C)$

On note $\ell^1(\C)$ l'ensemble des suites complexes dont la série associée est absolument convergente. Alors $\ell^1(\C)$ est un espace vectoriel normé, pour la norme $\norme{u}_1=\dsp\sum_{n=0}^{+\infty}|u_n|$.

On note $\ell^2(\C)$ l'ensemble des suites complexes $(u_n)$ telles que la série $\dsum |u_n|^2$ soit convergente. $\ell^2(\C)$ est un espace vectoriel préhilbertien complexe, pour le produit scalaire hermitien.
$ < u|v >=\dsp\sum_{n=0}^{+\infty}\overline{u_n}v_n$. La norme associée est notée $\norme{\phantom{u}}_2$, définie par : $\dsp\norme{u}_2=\sqrt{\dsp\sum_{n=0}^{\infty}|u_n|^2}$.

Soit $(u_n)$ et $(v_n)$ deux éléments de $\ell^2(\C)$. Alors $(u_nv_n)$ est élément de $\ell^1(\C)$, et on a l'inégalité de Cauchy-Schwarz : $$ \dsp\left|\dsp\sum_{n=0}^{+\infty}u_nv_n\right|\leq \norme{uv}_1\leq\norme{u}_2\cdot\norme{v}_2.$$

Remarque

$\ell^1(\C)\subset\ell^2(\C)$, l'inclusion étant stricte. De plus, les normes $\norme{\phantom{u}}_1$ et $\norme{\phantom{u}}_2$ ne sont pas équivalentes sur $\ell^1(\C)$.

Produit de Cauchy de deux séries

Soit $(u_n)_{n\in\N}$ et $(v_n)_{n\in\N}$ deux suites à valeurs complexes. On appelle série produit de Cauchy des séries de terme général $u_n$ et $v_n$ la série de terme général $w_n$ avec $$\forall n\in\N, \ w_n=\dsp\sum_{k=0}^n u_k v_{n-k}=\dsp\sum_{\underset{\scriptstyle p,q\in\N}{\scriptstyle p+q=n}}u_pv_q.$$

Si $\sum u_n$ et $\sum v_n$ sont absolument convergentes, alors $\sum w_n$ est absolument convergente, et de plus : $$\dsp\sum_{n=0}^{\infty} w_n=\left(\sum_{n=0}^{\infty}u_n\right)\left(\sum_{n=0}^{\infty} v_n\right).$$

Remarques

  • On peut montrer que la convergence absolu de l'une de deux séries ($\sum u_n$ ou $\sum v_n$) implique la convergence de le série produit $\sum w_n$.
  • On peut montrer aussi que si les trois séries $\dsum u_n$, $\dsum v_n$ et $\dsum w_n$ sont convergentes, alors $\dsum w_n= (\dsum u_n)(\dsum v_n)$.
  • La convergence de la série $\dsum w_n$ n'est plus assurée les deux séries $\dsum u_n$ et $\dsum v_n$ ne sont pas absolument convergentes. c.f. exemple ci-après.

Exemple

On considère la série $U=\dsum_{n\geq 1} \dfrac{(-1)^n}{\sqrt{n}}$, on sait que cette série est semi-convergente.
On considère le produit de Cauchy $W=(w_n)_{n\geq 1}$ de $U$ avec $U$.
Pour tout $n\geq 2$, on a (on pose $u_0=0$) $$w_n=\dsum_{k=0}^nu_ku_{n-k}=\dsum_{k=1}^{n-1}\dfrac{(-)^k(-1)^{n-k}}{\sqrt{k}\sqrt{n-k}}=(-1)^n\dsum_{k=1}^{n-1}\dfrac{1}{\sqrt{k(n-k)}}=\dfrac{(-1)^n}{n}\dsum_{k=1}^{n-1}\dfrac{1}{\sqrt{(k/n)(1-(k/n))}}$$ puisque $k/n\in ]0,1[$ alors $\sqrt{(k/n)(1-(k/n))}\leq \dfrac{1}{2}$, on en déduit alors, $$\abs{w_n}=\dfrac{1}{n}\dsum_{k=1}^{n-1}\dfrac{1}{\sqrt{(k/n)(1-(k/n))}}\geq \dfrac{2(n-1)}{n} \text{ donc } w_n \text{ ne tend pas vers } 0$$ Autrement dit, la série $\dsum w_n$ est divergente.


Soit $a\in ]-1,1[$. Montrer que la série $\dsum (n+1)a^n$ converge et calculer sa somme.

Correction

On considère la série $U=\dsum a^n$, on sait que c'est une série absolument convergente (puisque $\abs{a} < 1$). On note $W$ la série produit de Cauchy de $U$ par elle même. $$\forall n\geq 0,\quad w_n=\dsum_{k=0}^n u_ku_{n-k}=\dsum_{k=0}^n a^ka^{n-k}=\dsum_{k=0}^n a^n=(n+1)a^n.$$ On a $W$ est absolument convergente et $W=U^2$, i.e. $$\forall a\in ]-1,1[,\quad \dsum_{n=0}^\infty (n+1)a^n=\left(\dsum_{n=0}^\infty a^n\right)^2=\dfrac{1}{(1-a)^2}.$$

Application : l'exponentielle complexe

Pour tout $z\in \C$, la série $\dsp\sum_{n\in\N}\dfrac{z^n}{n!}$ est absolument convergente.

La somme de cette série se note $\textrm{exp}(z)$ ou $\ee^z$ : $$\forall z \in \C, \ \ee^z=\dsp\sum_{n=0}^{+\infty}\dfrac{z^n}{n!},$$ et s'appelle l' exponentielle du nombre complexe $z$.

Pour tous $(z,z')\in\C^2$ on a : $\ee^{z+z'}=\ee^z\cdot\ee^{z'}$.

Pour tout $z\in \C$, $\ee^z \neq 0$ et $\dfrac1{\ee^z}=\ee^{-z}$

Application: Formule de Stirling

La formule de Stirling permet de donner un équivalent de $n!$ lorsque $n$ tend vers $\infty$.
C'est le mathématicien français A. De Moivre qui a démontré la relation suivante : $$\dsp n!\underset{n\rightarrow\infty}{\thicksim} \,Cn^n\ee^{-n}\sqrt{ n}$$ avec $C$ une constante. La contribution de Stirling fut le calcul de la constante $C$.
La formule finale (c.f. théorème ci-après) est donnée par A. De Moivre en 1730.

Première approche On a la relation suivante: $\dsp \ln(n!)\underset{n\rightarrow\infty} {\thicksim} \,n\ln(n)-n$.

Deuxième approche On a la relation suivante: $\dsp \ln(n!)\underset{n\rightarrow\infty } {\thicksim} \,n\ln(n)-n+\dfrac{\ln(n)}{2}$.

On a la relation suivante: $\dsp n!\underset{n\rightarrow\infty } {\thicksim} \,n^n\ee^{-n}\sqrt{2\pi n}$.